(X-5)\(3x^2-14x-5) solve by polynomial long division

Answers

Answer 1

Answer: 3x+1

Step-by-step explanation: See attachment

(X-5)\(3x^2-14x-5) Solve By Polynomial Long Division

Related Questions

Add.

(3+x3+3x2)+(2x3−2−4x2)



Express the answer in standard form.

Enter your answer in the box.

Answers

Step-by-step explanation:

(3+3+3*2)+(2*3-2-4*2)

The value of the addition of the two polynomials is 3x³ - x² + 1.

What is addition of polynomial?

The addition of polynomials involves combining like terms of two or more polynomials to create a new polynomial. A polynomial is an expression consisting of variables, coefficients, and exponents, connected by addition and subtraction operations.

To add the given polynomials:

(3 + x³ + 3x²) + (2x³ - 2 - 4x²)

First, let's group the like terms:

(x³ + 2x³) + (3x² - 4x²) + (3 - 2)

Combine the coefficients of the like terms:

3x³ + 3x² - 4x² + 3 - 2

Simplify the expression:

3x³ - x² + 1

Therefore, the sum of the given polynomials is: 3x³ - x² + 1.

Learn more on addition of polynomial here;

https://brainly.com/question/10078818

#SPJ2

Someone help me with this math word questions

Answers

2) The three ways of naming an angle are by the letter at the vertex, by placing a number or variable inside the angle, or by two letters with the vertex letter placed in the middle.

3) [tex]\angle WXY, \angle YXZ, \angle WXZ[/tex]

4) [tex]X[/tex]

Give ur answer in standard form.
3 x 10^4 ÷6 x 10^-4

Answers

Answer:

5.0×10⁷

Step-by-step explanation:

(3÷6)×(10⁴÷10^-4)

(0.5)×(10⁸)

5.0×10^-1×10⁸

5.0×10⁷

[tex] \frac{3 \times {10}^{4} }{6 \times {10}^{ - 4} } \\ \\ \frac{1 \times {10}^{4 - ( - 4)} }{2 } \\ \\ \frac{ 1 \times {10}^{4 + 4} }{2} \\ \\ \frac{1 \times {10}^{8} }{2} \\ \\ 0.5 \times {10}^{8} .[/tex]


The sin (0) = - and lies in Quadrant III. Find the exact values of the sine
6'
and cosine of 20.

Answers

Answer:

Can you give exanation

I don't understand

A sequence is defined by the recursive function f(n + 1) = one-half(n). If f(3) = 9 , what is f(1) ?

Answers

The value of f(1) is 81

What is Recursive function?

Recursive Function is a function that repeats or uses its own previous term to calculate subsequent terms and thus forms a sequence of terms

Given:

f(n + 1) = 1/3* f(n)

f(3) = 9

let n=0

f(0+1) = 1/3 * f(0)

f(1) = 1/3 * f(0)

Let n=1

f(1+1) = 1/3 * 1/3*f(0)

f(2) = 1/9 * f(0)

let n=2

f(2+1) = 1/3 * 1/9* f(0)

f(3) = 1/27 * f(0)

Also, f(3) =9

So,

9 = 1/27 * f(0)

f(0) = 27 * 9

f(0) = 243

Now,

f(1) = 1/3 * f(0)

f(1) = 1/3/ 243

f(1) = 81

Learn more about this concept here:

https://brainly.com/question/14216181

#SPJ1

What function is shown in the graph above?

Answers

By definition of linear functions and the comparison with the attached figure, the function that represents the graph is y = (7/15) · x + 4, - 6 ≤ x ≤ 9.

What kind of function represents the graph?

Graphically speaking, linear functions represent lines and we see that the line seen in the figure presents two bounds, the points (- 6, 0) and (9, 7). Linear functions are characterized by slope and intercept:

y = m · x + b     (1)

Slope

m = (7 - 0)/[9 - (- 6)]

m = 7/15

Intercept

b = 4

By definition of linear functions and the comparison with the attached figure, the function that represents the graph is y = (7/15) · x + 4, - 6 ≤ x ≤ 9.

To learn more on linear functions: https://brainly.com/question/14695009

#SPJ1

A basketball coach recorded how many shots each of the 30 players at tryouts made in a set of 6 attempts.

Answers

Answer:

Step-by-step explanation:

what is the question?

Answer:

Median = A

Mean = B

Step-by-step explanation:

Khan Academy

how do i Solve this equation
√x+5-2=0

Answers

Answer:

9

Step-by-step explanation:

√x+5-2=0

√x+3=0

√x= -3

x=9   [(-3)*(-3) = 9]

Step 1 - Add 2 - √x + 5 = 2
Step 2 - Minus 5 - √x = -3
Step 3 - Square - x = -9

f(x) = x + 6 , find the ordered pair when x = -2.

(-2,4)

(2,-4)

(-2,-4)

(2,4)

Answers

-------------------------------------------------------------------------------------------------------------

Answer:  [tex]\textsf{Option A, (-2, 4)}[/tex]

-------------------------------------------------------------------------------------------------------------

Given:  [tex]\textsf{f(x) = x + 6}[/tex]

Find:  [tex]\textsf{f(-2)}[/tex]

Solution:  We need to plug in the value -2 into every variable x that we have in the expression and simplify to get the solution.

Plug in the values

[tex]\textsf{f(x) = x + 6}[/tex][tex]\textsf{f(-2) = -2 + 6}[/tex]

Simplify

[tex]\textsf{f(-2) = -2 + 6}[/tex][tex]\textsf{f(-2) = 4}[/tex]

Therefore, the ordered pair that would match would have a x-coordinate of -2 and a y-coordinate of 4 producing (-2, 4) which matches option A.

Carolyn bought two brands of socks. She bought a total of 5 pairs. One brand cost $2 each and the other brand cost $3 each. She spent $13. Find the number of pairs each brand of socks she bought

Answers

Answer:

Step-by-step explanation:

Declaration

Let Brand A number of 2.00 socks= x

Let Brand B number of 3.00 socks = y

Equations

Number of socks : x + y = 5

Cost of socks: 2x + 3y = 13

Solution

Multiply top equation by 2

2(x+ y = 5)

2x + 2y = 10

Write the new equation under the cost of socks equation

2x + 3y = 13

2x + 2y = 10       Subtract.

y = 3

She spent 3 dollars on 1 pair of socks.

Now she needs to calculate the number of 2.00 dollar socks

2x + 3y = 13

2x + 3*3 = 13                Combine the left

2x + 9 = 13                   Subtract 9 from both sides.

2x +9-9 = 13-9             Combine

2x = 4                          Divide by 2

2x/2 = 4/2

x = 2

Answer

#2 dollar socks = 2

#3 dollar socks = 3

Jose began the day with eight Magikarps. After spending the day at the lake, he ended up with sixteen Magikarps.

Find the Percent of increase
thats all she wrote

Answers

A percentage is a way to describe a part of a whole. The percentage increase of Magikarps is 100%.

What are Percentages?

A percentage is a way to describe a part of a whole. such as the fraction ¼ can be described as 0.25 which is equal to 25%.

To convert a fraction to a percentage, convert the fraction to decimal form and then multiply by 100 with the '%' symbol.

Jose began the day with eight Magikarps. After spending the day at the lake, he ended up with sixteen Magikarps. Therefore, The percentage increase of Magikarps is,

Percentage increase = (16-8)/8 × 100% = 100%

Hence, The percentage increase of Magikarps is 100%.

Learn more about Percentages:

https://brainly.com/question/6972121

#SPJ1

The correlation in error terms that arises when the error terms at successive points in time are related is termed _____.

Answers

Answer:

auto correlation is the answer to the question

4. Write the function of a linear equation that includes the
points (1,3) and (2,9).

Answers

Answer:

y = 6x - 3

Step-by-step explanation:

the equation of a line in slope- intercept form ( linear function ) is

y = mx + c ( m is the slope and c the y- intercept )

calculate m using the slope formula

m = [tex]\frac{y_{2}-y_{1} }{x_{2}-x_{1} }[/tex]

with (x₁, y₁ ) = (1, 3 ) and (x₂, y₂ ) = (2, 9 )

m = [tex]\frac{9-3}{2-1}[/tex] = [tex]\frac{6}{1}[/tex] = 6 , then

y = 6x + c ← is the partial equation

to find c substitute either of the 2 points into the partial equation

using (1, 3 ) , then

3 = 6 + c ⇒ c = 3 - 6 = - 3

y = 6x - 3 ← equation of linear function

Please help me out…

Polygon ____ and polygon ____ are similar to polygon 1.

Answers

Polygon 3 and 4 are similar to polygon 1.

Polygons will be called similar if they are exactly in the same shape, but can be in different sizes.

Here given, Some of the images are images of the polygon 1 from similarity transformations.

So we have to find the similar polygon from the given picture.

From picture it is clear that Polygon 2 is similar to polygon 1 as the size of polygon 2 has been decreased from polygon 1 but the shape of polygon 2 is the same as polygon 1 as we can their corresponding angles are also equal.

Also, polygon 3 is similar to polygon 1 as the size of polygon 3 has been decreased from polygon 1 but the shape of polygon 2 is the same as polygon 1 and reversed of polygon 1. So polygon 3 is of the same orientation as polygon 1 as we can their corresponding angles are also equal.

So from above, it is clear that Polygon 1 is similar to polygon 3 and 4 as they have the same shape but different sizes.

Therefore Polygon 3 and Polygon 4 are similar to polygon 1.

Learn more about similar polygon

here: https://brainly.com/question/1493409

#SPJ10

What is a 10% margin increase on 1,810.69?

Answers

Answer:

1991.759

Step-by-step explanation:

1810.69×%10=181.069

1810.69+181.069=1991.759

PLEASE ANSWER ASAP!!!!!! WILL GIVE BRAINLIEST!!!

Answers

Answer ya so  1 is the answer

Step-by-step explanation:

Answer:

1

Step-by-step explanation:

Given expression:

[tex]\sf \left(\dfrac{3a^{-2}b^6}{2a^{-1}b^5} \right)^2[/tex]

To find the value of the expression when a = 3 and b = -2, substitute these values into the expression:

[tex]\implies \sf \left(\dfrac{3(3)^{-2}(-2)^6}{2(3)^{-1}(-2)^5} \right)^2[/tex]

[tex]\textsf{Apply exponent rule} \quad a^{-n}=\dfrac{1}{a^n}[/tex]

[tex]\sf \implies \left(\dfrac{3\left(\dfrac{1}{3^2}\right)(-2)^6}{2\left(\dfrac{1}{3^1} \right)(-2)^5} \right)^2[/tex]

[tex]\sf \implies \left(\dfrac{3\left(\dfrac{1}{9}\right)(-2)^6}{2\left(\dfrac{1}{3} \right)(-2)^5} \right)^2[/tex]

[tex]\sf \implies \left(\dfrac{\left(\dfrac{3}{9}\right)(-2)^6}{\left(\dfrac{2}{3} \right)(-2)^5} \right)^2[/tex]

[tex]\sf \implies \left(\dfrac{\left(\dfrac{1}{3}\right)(-2)^6}{\left(\dfrac{2}{3} \right)(-2)^5} \right)^2[/tex]

[tex]\textsf{Apply exponent rule} \quad (-a)^n=a^n,\:\: \textsf{ if }n \textsf{ is even}[/tex]

[tex]\textsf{Apply exponent rule} \quad (-a)^n=-a^n,\:\: \textsf{ if }n \textsf{ is odd}[/tex]

[tex]\sf \implies \left(\dfrac{\left(\dfrac{1}{3}\right) (2^6)}{\left(\dfrac{2}{3} \right) (-(2^5))} \right)^2[/tex]

[tex]\sf \implies \left(\dfrac{\left(\dfrac{1}{3}\right) (64)}{\left(\dfrac{2}{3} \right) (-32)} \right)^2[/tex]

[tex]\sf \implies \left(\dfrac{\left(\dfrac{1 \times 64}{3}\right)}{\left(\dfrac{2 \times -32}{3} \right)} \right)^2[/tex]

[tex]\sf \implies \left(\dfrac{\dfrac{64}{3}}{\dfrac{-64}{3}} \right)^2[/tex]

When dividing fractions, flip the second fraction and multiply it by the first:

[tex]\implies \sf \left( \dfrac{64}{3} \times \dfrac {3}{-64} \right)^2[/tex]

[tex]\implies \sf \left( \dfrac{64 \times 3}{3 \times (-64)}\right)^2[/tex]

[tex]\implies \sf \left( \dfrac{192}{-192}\right)^2[/tex]

[tex]\implies \sf \left(-1\right)^2[/tex]

[tex]\textsf{Apply exponent rule} \quad (-a)^n=a^n,\:\: \textsf{ if }n \textsf{ is even}[/tex]

[tex]\sf \implies 1^2=1[/tex]

Please Help me!!!!!!!!!

Answers

Answer:

(2,-3) (3,-3)

Step-by-step explanation:

f(x) is y

x² - 5x + 3 = -3

x² - 5x + 6 = 0

ask yourself

which 2 numbers when multiplied give you

+6 and when added give you -5

they are -3 and -2

(x-3)(x-2)

x=3 x=2

when you put x=3 and x=2

in x² - 5x + 3 it's gives -3 which is y

(3,-3) (2,-3)

https://brainly.com/question/21836835

The vertex of this parabola is at (3-2). When the value is 4 the
y-value is 3. What is the coefficient of the squared expression in the
parabola's equation?
A. 7
B. 1
C. -1
D. 5

Answers

Answer:

D

Step-by-step explanation:

the equation of a parabola in vertex form is

y = a(x - h)² + k

where (h, k ) are the coordinates of the vertex and a is a multiplier

here (h, k ) = (3, - 2 ) , then

y = a(x - 3)² - 2

to find a substitute the point (4, 3 ) into the equation

3 = a(4 - 3)² - 2 ( add 2 to both sides )

5 = a × 1² = a

the coefficient of the squared expression is 5

8.
(05.04 LC)

The amount of money in tips earned by four restaurant servers waiting on 10 tables is represented by the following data sets.

Alyssa {3, 6, 2, 8, 12, 14, 5, 7, 7, 8}
Bryant {9, 2, 7, 50, 0, 5, 2, 8, 6, 8}
Camila {1, 9, 10, 3, 0, 12, 10, 9, 8, 2}
Devon {4, 2, 8, 15, 20, 7, 5, 0, 6, 2}

Which data set has the greatest interquartile range? (1 point)
Alyssa

Bryant

Camila

Devon

Answers

Camila's data set has the greatest interquartile range and the value of IQR = 8 option third is correct.

What is the range?

It is defined as the difference between the maximum value in the data set to the minimum value in the data set.

We have:

The amount of money in tips earned by four restaurant servers waiting on 10 tables is represented by the following data sets:

Alyssa {3, 6, 2, 8, 12, 14, 5, 7, 7, 8}

Bryant {9, 2, 7, 50, 0, 5, 2, 8, 6, 8}

Camila {1, 9, 10, 3, 0, 12, 10, 9, 8, 2}

Devon {4, 2, 8, 15, 20, 7, 5, 0, 6, 2}

The IQR for Alyssa:

IQR = Q3 - Q1

IQR = 8 - 5 = 3

The IQR for Bryant:

IQR = Q3 - Q1

IQR = 8 - 2 = 6

The IQR for Camila:

IQR = Q3 - Q1

IQR = 10 - 2 = 8

The IQR for Devon:

IQR = Q3 - Q1

IQR = 8 - 2 = 6

Thus, Camila's data set has the greatest interquartile range and the value of IQR = 8 option third is correct.

Learn more about the range here:

https://brainly.com/question/17553524

#SPJ1

if x + 3y = 25 write y in terms of x and also find the two solutions of this equation

Answers

Answer:

y= 25/3 - x/3

Step-by-step explanation:

y= 25/3 - x/3

(8.6x107)-(9.1x10-8)simplify

Answers

Answer:

  85,999,999.999 999 909

Step-by-step explanation:

The expression represents the difference of a relatively large number and one that is relatively small. That difference is approximately the value of the large number. The exact value requires 17 digits for its proper expression. Most calculators and spreadsheets cannot display this many digits.

Standard form

The numbers in standard form are ...

  86,000,000 = 8.6×10^7

  0.000000091 = 9.1×10^-8

Difference

Their difference is ...

  86,000,000 -0.000000091 = 85,999,999.999 999 909

In scientific notation, this is ...

  8.599 999 999 999 990 9×10^7

Given: l || m; ∠1 ∠3

Prove: p || q

Horizontal and parallel lines l and m are intersected by parallel lines p and q. At the intersection of lines l and p, the uppercase left angle is angle 1. At the intersection of lines q and l, the bottom right angle is angle 2. At the intersection of lines q and m, the uppercase left angle is angle 3.

Complete the missing parts of the paragraph proof.



We know that angle 1 is congruent to angle 3 and that line l is parallel to line m because
. We see that is congruent to by the alternate interior angles theorem. Therefore, angle 1 is congruent to angle 2 by the transitive property. So, we can conclude that lines p and q are parallel by the

.

Answers

The parts that are missing in the proof are:

It is given

∠2 ≅ ∠3

converse alternate exterior angles theorem

What is the Converse of Alternate Exterior Angles Theorem?

The theorem states that, if two exterior alternate angles are congruent, then the lines cut by the transversal are parallel.

∠1 ≅ ∠3 and l║m because we are: given

By the transitive property,

∠2 and ∠3 are alternate interior angles, therefore, they are congruent to each other by the alternate interior angles theorem.

Based on the converse alternate exterior angles theorem, lines p and q are proven to be parallel.

Therefore, the missing parts pf the paragraph proof are:

It is given∠2 ≅ ∠3converse alternate exterior angles theorem

Learn more about the converse alternate exterior angles theorem on:

https://brainly.com/question/17883766

#SPJ1

Answer:

it is given

angle 2

angle 3

converse alternate exterior angles theorem

Step-by-step explanation:

A company making tires for bikes is concerned about the exact width of its cyclocross tires. The company has a lower specification limit of 22.8 millimeters and an upper specification limit of 23.1 millimeters. The standard deviation is 0.19 millimeters and the mean is 22.9 millimeters. What is the process capability index for the process? Note: Round your answer to 4 decimal places.

Answers

The process capability index for the process is 0.1754.

How to calculate the index?

The first sided specification limit will be:

= (Upper specification limit - mean)/(3 × standard deviation)

= (23.1 - 22.9)/(3 × 0.19)

= 0.2/0.57

= 0.3508

The second sided specification limit will be:

= (22.9 - 22.8)/(3 × 0.19)

= 0.1/0.57

= 0.1754

The process capability index for the process is 0.1754 wine it's the lower value.

Learn more about capability index on:

brainly.com/question/15734839

#SPJ12

An architect is designing new housing structures for the primate section at the zoo. Her plan is shown below. Which animals will live in a building that is similar to the main primate house? A orangutans B chimpanzees C gibbons D gorillas

Answers

Answer:

A

Step-by-step explanation:

:p

Find the HCF of
x^4 y^2 and x^3 y^3

Answers

Here's your answer down here↓:

Step-by-step explanation:

x^3 - y^3 = (x - y)(x^2 + xy + y^2)

(x^4 - y^4) = (x^2 - y^2)(x^2 + y^2) = (x - y)(x + y)(x^2 + y^2)

Ok. So the factor (x-y) appears once in the top line and once in the second line. So we are going to take it the least amount of times.

So, the factor (x + y) appears in the top line zero times and in the second line one time so we will take it where it appears the least which is zero times so we are still at (x - y)

And it Same goes for the factors of (x^2 + y^2) and (x^2 + xy + y^2)

what is the equation?
3 + 5(2 + 7n) = 258

Answers

Answer:

n = 7

Explanation:

[tex]\rightarrow \sf 3 + 5(2 + 7n) = 258[/tex]

apply distributive method: a(b + c) = ab + ac

[tex]\rightarrow \sf 3 + 10 + 35n = 258[/tex]

simply add the numbers

[tex]\rightarrow \sf 13 + 35n = 258[/tex]

subtract both sides by 13

[tex]\rightarrow \sf 13-13 + 35n = 258-13[/tex]

simplify the following

[tex]\rightarrow \sf 35n = 245[/tex]

divide both sides by 35

[tex]\rightarrow \sf n = 7[/tex]

Answer:

n = 7

Step-by-step explanation:

1) Subtract 3 from both sides.

5(2 + 7n) = 258 - 3

2) Simplify 258 - 3 to 255.

5(2 + 7n) = 255

3) Divide both sides by 5.

2 + 7n = 255/5

4) Simplify 255/5 to 51.

2 + 7n = 51

5) Subtract 2 from both sides.

7n = 51 - 2

6) Simplify 51 - 2 to 49.

7n = 49

7) Divide both sides by 7.

n = 49/7

8) Simplify 49/7 yo 7.

n = 7

f(x)=x^2. which of these is g(x) ?

a. g(x)=(1/5x)^2
b. g(x)=5x^2
c.g(x)=(1/4x)^2
d.g(x)=1/5x^2

Answers

Answer:

a

Step-by-step explanation:

we know that g(x) means y therefore in this case let's suppose we have the coordinate (5;y) so we must find the corresponding y value

*DO THE TRIAL AND ERROR METHOD*

take any equation above and substitute the value of x of which is 5 and the corresponding value should be 1.

[tex]g(x) = (\frac{1}{5} x) {}^{2} [/tex]

The quadrilateral below is formed from a parallelogram and an
isosceles triangle.
Calculate the size of angle VQU.

Answers

Answer:

VQU =58°

Step-by-step explanation:

angle QUV is 61 because of F angles (I don't know what they call them in ur country)

triangle QUV is an isosceles triangles so base angles are the same

61+61=122

180-122=58

[tex](1/8)^-^3^a=512^3^a[/tex]

Answers

The equation [tex](\frac18)^{-3a} = 512^{3a}[/tex] as no solution

How to solve the equation?

The equation is given as:

[tex](\frac18)^{-3a} = 512^{3a}[/tex]

Express the equation as a base of 2

[tex](2^{-3})^{-3a} = 2^{9*3a}[/tex]

Evaluate the products

[tex]2^{9a} = 2^{27a}[/tex]

Cancel out the common base

9a = 27a

The above equation is not true because [tex]9a \ne 27a[/tex]

Hence, the equation [tex](\frac18)^{-3a} = 512^{3a}[/tex] as no solution

Read more about equations at:

https://brainly.com/question/2456547

#SPJ1

During a catered lunch, an average of 4 cups of tea are poured per minute. The lunch will last 2 hours. How many gallons of tea should the caterer bring if there are 16 cups in one gallon?

Answers

Answer:

30

Step-by-step explanation:

4 cups x 60 hours a min x 2 hours of lunch ÷ 16 cups per gallon = 30 gallons

The answer is 30 gallons.

Firstly, convert the hours to minutes so you know how many minutes you're working with. 2*60=120 minutes.

Then, multiply 120 with 4 to get the number of tea (in cups) that is poured. 120*4=480 cups.

Finally, divide 480 cups with 16 cups to find out the number of gallons poured. 480/16 equals 30 gallons.

Hope this helped.

Other Questions
Which perspective is most concerned with how individuals interpret their experiences?. Solve the following system to find the point(s) of intersection. y = x - 2x - 3y = 2x - 3 The study of the effects of drugs on the mind and brain is:A. transcranial magnetic stimulation therapy.B. electroconvulsive therapy.C. psychopharmacology.D. psychotherapy.Ask my instructor Solve -6 < 2x + 4 16 HELLPPLet n be the middle number of three consecutive even integers. Write an expression for the sum of these integers.sum of the integers? Precepts about dealing with financial crises that most economists would sign onto are: Justify the following statement - 'You can't always tell if a person has an intellectual disability by looking at them.' Please help me solve this questionn as fast as possible oofff Which of the following statements is correct? An externality is a situation where a project would have an adverse effect on some other part of the firms overall operations. If the project would have a favorable effect on other operations, then this is not an externality. An example of an externality is a situation where a bank opens a new office, and that new office causes deposits in the banks other offices to decline The NPV method automatically deals correctly with externalities, even if the externalities are not specifically identified, but the IRR method does not. This is another reason to favor the NPVBoth the NPV and IRR methods deal correctly with externalities, even if the externalities are not specifically identified. However, the payback method does notIdentifying an externality can never lead to an increase in the calculated NPV Fill in the blank. Micheal Jordan es mas alto (blank) tu. Answer options: A. queB. deC. both que and deD. como Which of the following statements about psychological constructs is true?A.Psychological constructs are easy to define and measure.B.Psychological constructs represent concepts that cannot be observed or measured directly.C.Psychological constructs cannot be researched in psychology because they are unobservable.D.Psychological constructs represent measurable overt behavior.Please select the best answer from the choices provided The table below shows the results of a survey that asked 1052 adults from a certain country if they favored or opposed a tax to fund education. A person is selected atrandom. Complete parts (a) through (c).(a) Find the probability that the person opposed the tax or is female.P(opposed the tax or is female) =(Round to the nearest thousandth as needed.)(b) Find the probability that the person supports the tax or is male.P(supports the tax or is male) = (Round to the nearest thousandth as needed.)(c) Find the probability that the person is not unsure or is female.P(is not unsure or is female)= (Round to the nearest thousandth as needed.) Neither n2 nor o2 are greenhouse gases because GOD PLEASE HELP!Which of these is NOT a key factor that contributed to the financial crisis that started in 2007?bond rating agencies giving CDOs very low ratings, discouraging investors to investhighly leveraged investment accounts that magnified losses when bond prices decreasedexcessive use of subprime mortgages to borrowers with poor creditbanks being incentivized to sell risky mortgages as mortgage-backed securities Is the point (11, 15) on the circle defined by (x-9) + (y-15) = 4?O YesO No What was the purpose of the Munich conference? Which is true regarding the graphed function f(x)? f(O) = 3 f(5) = - 1 f(3) = 2O f(2) = -2 If a utility burned 7.57 million tons of coal that was 2.00% sulfur by weight, how many tons of sulfur dioxide were emitted? Answer in scientific notation. find x if 2+x = -2/3 Marion is going to the hospital for a triple bypass operation. she will have general anesthesia, an intravenous catheter, surgical wounds, and a urinary catheter. which healthcare-associated infection is she at greatest risk for contracting